Download as pdf or txt
Download as pdf or txt
You are on page 1of 37

https://www.pdfnotes.

co/

O
C
S.
TE
O
FN
D
.P
W
W
W
https://www.pdfnotes.co/

DO NOT OPEN THIS TEST BOOKLET UNTIL YOU ARE ASKED TO DO SO

PPP-PTS-4273/082023/04

GENERAL STUDIES

POLITY - 4
Time Allowed: One Hour Maximum Marks: 100

INSTRUCTIONS

1. IMMEDIATELY AFTER THE COMMENCEMENT OF THE EXAMINATION, YOU SHOULD CHECK THAT
THIS TEST BOOKLET DOES NOT HAVE ANY UNPRINTED OR TORN OR MISSING PAGES OR ITEMS,
ETC. IF SO, GET IT REPLACED BY A COMPLETE TEST BOOKLET.
2. Please note that it is the candidate’s responsibility to encode and fill in the Roll Number carefully
without any omission or discrepancy at the appropriate places in the OMR Answer Sheet. Any
omission/discrepancy will render the Answer Sheet liable for rejection.
3. You have to enter your Roll Number on the test booklet in the Box provided
alongside. DO NOT write anything else on the Test Booklet.
4. This Test Booklet contains 50 items (questions). Each item is printed in English. Each item comprises four
responses (answers). You will select the response which you want to mark on the Answer Sheet. In case you
feel that there is more than one correct response, mark the response which you consider the best. In any case,
choose ONLY ONE response for each item.
5. You have to mark all your responses ONLY on the separate Answer Sheet provided. See directions in the
Answer Sheet.
6. All items carry equal marks.
7. Before you proceed to mark in the Answer Sheet in response to various items in the Test Booklet, you have to
fill in some particulars in the Answer Sheet as per instructions sent to you with your Admission Certificate.
8. After you have completed filling in all your responses on the Answer Sheet and the examination has concluded,
you should hand over to the Invigilator only the Answer Sheet. You are permitted to take away with you the
Test Booklet.
O
9. Sheets for rough work are appended in the Test Booklet at the end.
.C
10. Penalty for wrong answers:
S

THERE WILL BE PENALTY FOR WRONG ANSWERS MARKED BY A CANDIDATE IN THE OBJECTIVE
TE

TYPE QUESTION PAPERS.


(i) There are four alternatives for the answer to every question. For each question for which a wrong answer
has been given by the candidate, one-third of the marks assigned to that question will be deducted as
O

penalty.
FN

(ii) If a candidate gives more than one answer, it will be treated as a wrong answer even if one of the given
answers happens to be correct and there will be same penalty as above to that question.
D

(iii) If a question is left blank, i.e., no answer is given by the candidate, there will be no penalty for that
.P

question.

DO NOT OPEN THIS TEST BOOKLET UNTIL YOU ARE ASKED TO DO SO


W
W
W
https://www.pdfnotes.co/

1. Regarding the evolution of the third tier How many of the above statements are
governance in India, consider the following correct?
statements: (a) Only One
1. In 1957, the Ashok Mehta Committee (b) Only Two
recommended the establishment of a
three-tier structure of Panchayati Raj (c) All three
across states. (d) None
2. The Balwant Rai Mehta Committee
recommended mandatory quotas for the 4. Which one of the following Committee
recommended a three-tier structure of self-
Scheduled Castes and Scheduled Tribes.
governance in the tribal areas?
Which of the statements given above is/are (a) Ashok Mehta Committee
correct?
(b) Bhuria Committee
(a) 1 only
(c) Balwant Rai Mehta Committee
(b) 2 only
(d) G V K Rao Committee
(c) Both 1 and 2

RE
(d) Neither 1 nor 2 5. Consider the following provisions under PESA
Act:

2. With reference to the 73rd Amendment Act, 1. Gram sabha at the para, majra and tola
levels
consider the following statements:
2. Gram sabha to protect the traditions,
1. The Act unveiled three-tier rural beliefs and culture of the tribal
institutions - Gram
O
Panchayats, communities
Panchayat Samitis, and Zilla Parishad.
3. Gram sabha to manage and protect
2. The Act mandated one-third of the seats common properties based on their
at all levels of panchayats are reserved traditional systems of management and
SC
for women. protection

3. It mandated the formation of state finance 4. The administration to seek permission


from the gram sabha in case of land
commissions and provision for grants-in-
acquisition
aid from state governments to PRIs.
Select the correct answer using the codes given
How many of the above statements are below.
correct?
(a) 4 only
GS

(a) Only One


(b) 1 and 2 only
(b) Only Two
(c) 2 and 3 only
(c) All three (d) 1, 2, 3 and 4
(d) None
O
6. Regarding the sources of revenues of Urban
.C
Local Bodies (ULBs), consider the following:
3. Consider the following statements about 74th
Amendment Act: 1. Collection from tax and non-tax sources
S

as assigned to them under Municipal


TE

1. Under the Act, the creation of a Finance act.


Commission to determine the sharing of
taxes, duties, and funds from the State 2. Devolution of shared taxes and duties
as per recommendation of State Finance
O

Consolidated Fund.
Commission
FN

2. The government passed the Panchayat 3. Grants-in-aid from the State


Extension to Tribal Areas (PESA) Act Government
under 73rd Amendment Act.
D

4. Grants-in-aid from the Government


3. The Act reserves 50 percent of local body of India under Centrally Sponsored
.P

seats for ST community. Schemes


W

2 PPP-PTS-4273/082023/04
W
W
https://www.pdfnotes.co/

Select the correct answer using the codes given 2. It encourages citizen participation in the
below. decision-making process.
(a) 1, 2, 3 and 4 3. It is responsible for providing various
essential services like water supply,
(b) 1 only sanitation, education, healthcare,
(c) 2 only infrastructure development, and more.
How many of the above statements are
(d) 3 and 4 only
incorrect?

7. Consider the following functions and powers (a) Only One


enshrined under 11th Schedule of Indian (b) Only Two
Constitution:
(c) All three
1. Agriculture, including agricultural (d) None
extension
2. Land improvement, implementation of 10. With reference to the State Election Commission,
land reforms, land consolidation and soil consider the following statements:
conservation 1. It is a constitutional body established

RE
3. Minor irrigation, water management and under the provisions of the Indian
watershed development Constitution.

4. Maintenance of community assets 2. It is under the control of the state


government.
Select the correct answer using the codes given
3. It has power to conduct elections to the
below. O local bodies, such as urban and rural
(a) 1, 2 and 3 only local bodies within the state.
(b) 3 and 4 only 4. The Chief State Election Commissioner
and other Election Commissioners are
(c) 1, 2, 3, and 4 appointed by the Governor of the state.
SC
(d) None of them How many of the above statements are
correct?
8. With the reference to the XIIth Schedule of the (a) Only One
Indian Constitution, consider the following
(b) Only Two
statements:
(c) Only three
1. It was added by the 73rd Amendment Act
of 1992. (d) All four
GS

2. It aims to strengthen local self-government


11. Consider the following statements regarding
and promote effective and efficient urban
the State Election Commission:
planning and development.
1. It is responsible for the delimitation of
Which of the statements given above is/are constituencies for local body elections.
O
correct?
2. The Commission has no authority to
.C
(a) 1 only enforce the Model Code of Conduct during
(b) 2 only the election period.
S

3. It may provide advice to the state


TE

(c) Both 1 and 2


government on matters related to the
(d) Neither 1 nor 2 electoral reforms.
O

Which of the statement given above is/are


9. About the Local Governance in India, consider correct?
FN

the following statements: (a) 1 only


1. Local governance ensures the (b) 2 and 3 only
D

decentralization of power from the


central and state governments to the (c) 1 and 3 only
.P

local bodies. (d) 1, 2 and 3


W

PPP-PTS-4273/082023/04 3
W
W
https://www.pdfnotes.co/

12. About Finance Commission, consider the 3. The union cannot give directives to the
following statements: state pertaining to the construction and
maintenance within the state.
Statement-I:
How many of the above statements are
Our Constitution has made provision for the
creation of Finance Commission under article correct?
280. (a) Only one
Statement-II: (b) Only two
It’s objective to recommend devolution of (c) All three
resources from the Center to the States
through Tax-Sharing and Grants-in-Aid. (d) None

Which one of the following is correct in respect


of the above statements? 15. About financial relations between the
union and the states, consider the following
(a) Both Statement-I and Statement-II are statements:
correct and Statement-II is the correct
explanation for Statement-I 1. It is derived from the Government of

RE
India Act of 1909.
(b) Both Statement-I and Statement-II
are correct and Statement-II is not the 2. The seventh schedule of the Constitution
correct explanation for Statement-I provides for specific entries reserved for
the union and the states for imposing
(c) Statement-I is correct but Statement-II
taxes.
is incorrect
Which of the statements given above is/are
(d)
O
Statement-I is incorrect but Statement-
II is correct
correct?
(a) 1 only
13. With respect to the role of State Finance (b) 2 only
SC
Commission, consider the following:
(c) Both 1 and 2
1. Assessing Local Body Finances
(d) Neither 1 nor 2
2. Reviewing Financial Devolution
3. Rationalization of Local Taxes and Fees 16. With reference to the Comptroller and Auditor
4. Reporting to the Governor General of India (CAG), consider the following
statements:
GS

How many of the above-mentioned are


correct? 1. CAG is the sole authority prescribed
in the Constitution entrusted with the
(a) All four responsibility of audit of accounts of the
(b) Only one Union.

(c) Only two 2. CAG audits maintenance of provident


O
funds accounts of State Government
.C
(d) Only three employees.
S

3. He is not liable to do performance audit


14. With respect to the administrative relations
for Companies and Corporations.
TE

between Centre and States, consider the


following statements: How many of the statements given above are
correct?
O

1. The Executive Power of the Union


extends to giving of direction to the state (a) Only one
FN

under Article 256 for their compliance.


(b) Only two
2. Parliament is entitled to use the state
D

machinery for the enforcement of the (c) All three


union laws, and confer powers and
.P

(d) None
entrust duties to the state.
W

4 PPP-PTS-4273/082023/04
W
W
https://www.pdfnotes.co/

17. Consider the following statements: Select the correct answer from the code given
below:
1. The Attorney General of India can
participate in the joint sitting of the (a) 1, 2 and 3 only
Parliament for his/her own removal.
(b) 2 only
2. The first Attorney General of India was
M. C. Setalvad. (c) 1 and 3 only

3. The Attorney general is not entitled (d) None


under the rules for ‘Office of Profit’ in the
Constitution. 20. Consider the following statements:
4. He is the ex-officio member of the Law
1. Election Commission is the only electoral
commission of India.
management body (EMB) in India.
How many of the statements given above are
incorrect? 2. As per the independent model of electoral
management, members of Electoral
(a) Only one management bodies (EMBs) are not

RE
(b) Only two accountable to any government ministry
or department.
(c) Only three
3. A national population or statistics bureau
(d) All four that produces electoral registers as part
of the general process of population
18. Consider the following statements: O registration is not considered to be an
1. Union Public service commission (UPSC) EMB.
is authorized to give advice to the How many of the statements given above are
Government on any matter referred by correct?
the President of India.
SC
(a) Only one
2. The formation of a permanent commission
for civil services was brought in the (b) Only two
Indian Councils Act of 1919.
(c) All three
Which of the statements given above is/are
correct? (d) None
GS

(a) 1 only
21. Regarding the Finance commission of India,
(b) 2 only consider the following statements:
(c) Both 1 and 2 1. Finance commission can recommend the
distribution of rural local bodies’ grants,
(d) Neither 1 nor 2
O
only to Gram Panchayats.
.C
19. With reference to Joint Public service 2. Finance commission can suggest the
Commission (JPSC), which of the statements share of taxes between Centre and the
S

given below are not correct? States.


TE

1. The President appoints the chairman of


Which of the statements given above is/are
a JPSC.
correct?
O

2. It is not a constitutional body.


(a) 1 only
FN

3. The First Joint Public service commission


of formed in 1966. (b) 2 only
D

4. It was taken from Government of India (c) Both 1 and 2


.P

Act, 1935.
(d) Neither 1 nor 2
W

PPP-PTS-4273/082023/04 5
W
W
https://www.pdfnotes.co/

22. Consider the following statements: How many of the statements given above are
correct?
1. The GST Council consists of the Union
Finance Minister and representatives (a) Only One
from all States and Union Territories.
(b) Only two
2. The GST council makes decisions on
(c) Only three
various aspects of GST, including tax
rates, exemptions, and administrative (d) All four
procedures.
3. GST Council Secretariat is responsible 25. Consider the following statements:
for conducting meetings and setting up
Statement I:
the agenda for those meetings.
4. The Finance Minister is the ex-officio Constitution of India contains provisions to
Secretary of GST Council. safeguard rights of minorities.

How many of the statements given above are Statement II:


correct? National Commission for Minorities (NCM)
(a) Only one notifies only Muslims, Christians, Sikhs,
Buddhists and Zoroastrians (Parsis) as

RE
(b) Only two minorities.
(c) Only three Which one of the following is correct in respect
(d) All four of the above statements?
(a) Both Statement-I and Statement-II are
23. With reference to National Commission for correct and Statement-II is the correct
Scheduled Castes, consider the following explanation for Statement I
statements:
O (b) Both Statement-I and Statement-II
1. It is a statutory body established to are correct and Statement-II is not the
protect rights of Scheduled Caste and correct explanation for Statement-I
Anglo Indian community.
(c) Statement-I is correct but Statement-II
SC
2. The economic and social development is incorrect
wing of the commission deals with cases
related to minimum wages for SCs in the (d) Statement-I is incorrect but Statement-
country. II is correct

Which of the statements given above is/are


incorrect? 26. With reference to administration of schedule
areas in India, consider the following
(a) 1 only statements:
GS

(b) 2 only 1. The Governor can increase or decrease


(c) Both 1 and 2 the area of any Scheduled Area in a
State.
(d) Neither 1 nor 2
2. The Governor is authorised to make O
regulations for the peace and good
24. Consider the following statements:
government of any Scheduled area in a
.C
1. The National Perspective Plan for State.
Women (1988-2000) recommended the
3. The President is empowered to direct
S

constitution of a Commission for Women


that any particular act of Parliament or
in India.
TE

the state legislature does not apply to a


2. The National commission for Women scheduled area.
(NCW) deals with the issues of NRI
How many of the above statements are
O

martial disputes.
correct?
3. Suo Motu cognizance can be taken for
FN

information received by the NCW related (a) Only one


to violence against women. (b) Only two
D

4. Legal Cell of NCW reviews the (c) All three


constitutional and legal safeguards
.P

provided to the Women. (d) None


W

6 PPP-PTS-4273/082023/04
W
W
https://www.pdfnotes.co/

27. Consider the following statements: How many of the above statements is/are
criteria for declaring any area as a Scheduled
Statement-I: Area under the Fifth Schedule?
In order to address the welfare and progress of (a) Only one
the scheduled tribes, each state with scheduled
(b) Only two
areas is required to establish a Tribes Advisory
Council. (c) All three

Statement-II: (d) All Four

In states where scheduled tribes exist but 30. Consider the following statements regarding
scheduled areas are absent, the President can the 5th Schedule Areas:
direct the establishment of a Tribes Advisory
1. Assam, Meghalaya, Tripura and Mizoram
Council.
are exempted from the 5thSchedule of the
Which one of the following is correct in respect Indian Constitution.
of the above statements? 2. Tribal habitations in few states like
(a) Both Statement-I and Statement-II are Kerala, Tamil Nadu, Karnataka and
correct and Statement-II is the correct West Bengal are covered under the fifth

RE
schedule.
explanation for Statement-I
3. At present, 10 States have Fifth Schedule
(b) Both Statement-I and Statement-II Areas.
are correct and Statement-II is not the
correct explanation for Statement-I How many of the above statements are
correct?
(c) Statement-I is correct but Statement-II
(a) Only one
is incorrect
(d)
O
Statement-I is incorrect but Statement-
II is correct
(b)
(c)
Only two
All three
(d) All Four
SC
28. With reference to administration of tribal
areas, consider the following statements: 31. With reference to the features of administration
contained in the Sixth Schedule, consider the
1. Tribal areas are autonomous districts following statements:
but not independent of state control.
Statement-I:
2. The Governor can split an autonomous The district and regional councils within their
district into multiple regions for diverse territorial jurisdictions can constitute village
GS

tribes. councils or courts for trial of suits and cases


Which of the statements given above is/are between the tribes.
correct? Statement-II:
(a) 1 only The jurisdiction of high court over suits
and cases between the tribes is specified
(b) 2 only
O
by the Governor.
(c) Both 1 and 2
.C
Which one of the following is correct in respect
(d) Neither 1 nor 2 of the above statements?
S

(a) Both Statement-I and Statement-II are


TE

29. Consider the following: correct and Statement-II is the correct


1. Preponderance of tribal population explanation for Statement-I
O

(b) Both Statement-I and Statement-II


2. Compactness and reasonable size of the
are correct and Statement-II is not the
area
FN

correct explanation for Statement-I


3. Viability as an administrative entity, (c) Statement-I is correct but Statement-II
such as a district, block, or taluk is incorrect
D

4. Economic backwardness in comparison (d) Statement-I is incorrect but Statement-


.P

to neighbouring areas II is correct


W

PPP-PTS-4273/082023/04 7
W
W
https://www.pdfnotes.co/

32. The Constitution, under Sixth Schedule, How many of the above statements are
contains special provisions for the correct?
administration of tribal areas in which of the
(a) 1 only
following states?
(b) 2 only
1. Tripura
(c) Both 1 and 2
2. Manipur
(d) Neither 1 nor 2
3. Assam
4. Meghalaya 35. Consider the following statements regarding
the Bodoland Territorial Council:
5. Nagaland
1. The Council was created under the 5th
6. Mizoram Schedule of the Indian Constitution
Select the correct answer using the code given 2. A Speaker leads the Council, while a
below: Chief Executive Member chairs the
executive committee.
(a) 1, 2, 3 and 4 only
3. The executive and legislative powers
(b) 3, 4, 5 and 6 only of the Council are derived from Sixth

RE
(c) 1, 3, 4 and 6 only Schedule of the Constitution.
How many of the above statements are
(d) 1, 2, 3, 4, 5 and 6
correct?
(a) Only one
33. Consider the following statements regarding
Inner Line Permit (ILP): (b) Only two
(c) All three
Statement-I:
O
Manipur, Mizoram, and Nagaland are the (d) None
states that require a permit.
36. Consider the following statements regarding
Statement-II:
SC
‘Development Councils’ in Assam:
It is obligatory for Indian citizens from outside 1. Government of Assam has created 33
those states to obtain a permit for entering the Development Councils to decentralize
protected state. power and responsibility.
Which one of the following is correct in respect 2. The main function of these Council bodies
of the above statements? is to formulate various developmental
schemes and to execute these in priority
(a) Both Statement-I and Statement-II are
and need base manner.
GS

correct and Statement-II is the correct


explanation for Statement-I 3. The creation of each of the Development
Council was made by separate Gazetted
(b) Both Statement-I and Statement-II Notification.
are correct and Statement-II is not the
correct explanation for Statement-I How many of the above statements are O
correct?
(c) Statement-I is correct but Statement-II
.C
(a) Only one
is incorrect
(b) Only two
(d) Statement-I is incorrect but Statement-
S

II is correct (c) All three


TE

(d) None
34. With reference to ‘Tribal Sub-Plan’, consider
the following statement:
O

37. Consider the following statements regarding


1. It was established to promote the the Legislative relations between the Centre
FN

development of areas with a significant and States:


tribal concentration. Statement-I:
D

2. It was renamed as Scheduled Tribe Both the Parliament and the State legislatures
Component (STC) by Ministry of Tribal can make laws on subjects given in the
.P

Affairs. Concurrent list.


W

8 PPP-PTS-4273/082023/04
W
W
https://www.pdfnotes.co/

Statement-II: How many of the above statements are


correct?
In case of conflict between the law of the State
and Union law on a subject in the Concurrent (a) Only one
list, the law of the Parliament prevails. (b) Only two
Which one of the following is correct in respect (c) Only three
of the above statements?
(d) All four
(a) Both Statement-I and Statement-II are
correct and Statement-II is the correct 40. Which of the following statements is/are
explanation for Statement-I incorrect?
(b) Both Statement-I and Statement-II 1. NITI Aayog has been constituted to
are correct and Statement-II is not the actualise the important goal of cooperative
correct explanation for Statement-I federalism and to enable good governance
in India.
(c) Statement-I is correct but Statement-II
is incorrect 2. NITI has established the Sustainable
Action for Transforming Human Capital

RE
(d) Statement-I is incorrect but Statement- (SATH) programme to promotion of
II is correct cooperative federalism among the states.
Select the correct answer using the code given
38. With respect to the Inter-State Councils, below.
consider the following statements:
(a) 1 only
1. The council was established on
recommendations
Commission.
of
O
Sarkaria (b)
(c)
2 only
Both 1 and 2
2. The councils empowered to investigate (d) Neither 1 nor 2
SC
and discuss subjects of common interest
between the Union and states, or among 41. Consider the following statements:
states.
Statement-I:
Which of the statements given above is/are
The term “emergency” refers to an
correct? unanticipated turn of circumstances that
(a) 1 only requires government bodies to act immediately
within their jurisdiction.
GS

(b) 2 only
Statement-II:
(c) Both 1 and 2
In situations of emergency, all civil rights -
(d) Neither 1 nor 2 except Articles 20 and 21 of the Constitution
of India suspended inside the state or nation.
O
39. In the context of the Zonal Councils, consider
Which one of the following is correct in respect
.C
the following pairs: of the above statements?
1. Northern Zonal Council - Himachal
S

(a) Both Statement-I and Statement-II are


Pradesh, Jammu & Kashmir correct and Statement-II is the correct
TE

explanation for Statement-I


2. Central Zonal Council - Chhattisgarh
and Uttarakhand (b) Both Statement-I and Statement-II
O

are correct and Statement-II is not the


3. The Eastern Zonal Council - Orissa and correct explanation for Statement-I
FN

Sikkim
(c) Statement-I is correct but Statement-II
4. Western Zonal Council - Union Territories is incorrect
D

of Daman & Diu and Dadra & Nagar (d) Statement-I is incorrect but Statement-
.P

Haveli II is correct
W

PPP-PTS-4273/082023/04 9
W
W
https://www.pdfnotes.co/

42. Consider the following statements regarding How many of the above statements are
the National Emergency in India: correct?

1. Article 356 states that the President can (a) Only one
proclaim an emergency when the territory (b) Only two
of India is under invasion, foreign
(c) All three
encroachment, or internal insurrection.
(d) None
2. The nation’s first emergency was
proclaimed during the 1962–1968
45. Consider the following statements:
confrontation with China.
1. Regional Bodies of NITI Aayog consists
Which of the statements given above is/are of Chief Ministers of all States which is
correct? summoned by the Prime Minister.
(a) 1 only 2. NITI Aayog is an advisory body.
(b) 2 only 3. Part-time members in NITI Aayog are
nominated on rotational basis.
(c) Both 1 and 2

RE
How many of the statements given above are
(d) Neither 1 nor 2 correct?
(a) Only one
43. Regarding the 42nd Amendment Act in India,
consider the following statements: (b) Only two

1. It introduced Fundamental Duties in the (c) All three


Constitution. (d) None
O
2. The Preamble of the Indian Constitution
enshrines the principles of socialism, 46. Regarding National Human Rights
Commission (NHRC), consider the following
secularism, and integrity under this
statements:
SC
amendment.
1. It has been set up by an Act of Parliament
3. The amendment declared that Parliament for the protection and promotion of
had the power to amend any part of the human rights.
Constitution excluding Fundamental
2. The Commission is responsible for
Rights.
spreading of human rights awareness
How many of the above statements are amongst the masses at National and
international level.
GS

correct?
Which of the statements given above is/are
(a) Only one
correct?
(b) Only two (a) 1 only
(c) All three (b) 2 only
O
(d) None (c) Both 1 and 2
.C
(d) Neither 1 nor 2
44. In the context of the Financial Emergency in
S

India, consider the following statements:


47. Consider the following:
TE

1. Article 360 gives authority to the


Statement I:
President of India to declare a financial
O

emergency. The State Information Commission conducts


investigations and renders judgments on
FN

2. The Supreme Court can review the appeal.


declaration of Financial Emergency.
Statement II:
D

3. A proclamation of financial emergency


must be approved by both the Houses of State Information Commission is a statutory
.P

body.
Parliament.
W

10 PPP-PTS-4273/082023/04
W
W
https://www.pdfnotes.co/

Which one of the following is correct in respect 49. Which of the following statements is/are
of the above statements? correct?

(a) Both Statement-I and Statement-II are 1. Article 352 was used by Union government
correct and Statement-II is the correct to declare the Emergency.
explanation for Statement I 2. The 44th Amendment Act of 1978 required
(b) Both Statement-I and Statement-II a six-month review of the emergency
are correct and Statement-II is not the declaration.
correct explanation for Statement-I 3. The National Emergency may stop the
enforcement of Articles 20 and 21 rights.
(c) Statement-I is correct but Statement-II
is incorrect Select the correct option using the codes given
below.
(d) Statement-I is incorrect but Statement-
II is correct (a) 3 only
(b) 1 and 2 only
48. With reference to Central Vigilance
Commission (CVC), which of the statements (c) 2 and 3 only
given below is not correct? (d) 1, 2 and 3

RE
1. The CVC was set up by the Government
in line with the Special Police 50. Consider the following:
Establishment. Counter-terrorism operations
2. In case of legal actions taken, an Probing cross-border crimes
Independent External Monitor is
appointed by Chief Vigilance Officers to Dealing with counterfeit currency cases
examine complaints received.
O
Select the correct answer from the code given
Combating cybercrimes
How many of the above-mentioned are correct
below: about National Investigation Agency (NIA)?
SC
(a) 1 only (a) All four
(b) 2 only (b) Only one
(c) Both 1 and 2 (c) Only two
(d) Neither 1 nor 2 (d) Only three
GS

™™™™™

O
S .C
TE
O
FN
D
.P
W

PPP-PTS-4273/082023/04 11
W
W
https://www.pdfnotes.co/

IAS Prelims - 2024


PPP-PTS-4273/082023/04

GENERAL STUDIES

POLITY - 4

Answer Key

Q. 1 (d) Q. 11 (c) Q. 21 (b) Q. 31 (b) Q. 41 (b)

Q. 2 (c) Q. 12 (a) Q. 22 (c) Q. 32 (c) Q. 42 (b)

Q. 3 (b) Q. 13 (a) Q. 23 (c) Q. 33 (d) Q. 43 (b)

Q. 4 (b) Q. 14 (b) Q. 24 (c) Q. 34 (a) Q. 44 (c)

Q. 5 (d) Q. 15 (b) Q. 25 (c) Q. 35 (b) Q. 45 (b)


O
.C
Q. 6 (a) Q. 16 (b) Q. 26 (a) Q. 36 (c) Q. 46 (c)
S

Q. 7 (c) Q. 17 (c) Q. 27 (b) Q. 37 (b) Q. 47 (c)


TE

Q. 8 (b) Q. 18 (a) Q. 28 (c) Q. 38 (c) Q. 48 (c)


O

Q. 9 (d) Q. 19 (d) Q. 29 (d) Q. 39 (d) Q. 49 (b)


FN

Q. 10 (c) Q. 20 (c) Q. 30 (b) Q. 40 (d) Q. 50 (a)


D
.P
W
W
W
https://www.pdfnotes.co/

1. Correct option: (d) 2. Correct option: (c)


Explanation: Explanation:
 Statement 1 is incorrect: In 1957,
the Balwant Rai Mehta Committee  All statements are correct
recommended the establishment of a three- Supplementary Notes:
tier structure of Panchayati Raj across
states. 73rd Amendment Act
 Statement 2 is incorrect: The Ashok  After decades of struggles and half
Mehta Committee recommended mandatory starts, finally, the Congress-led coalition
quotas for the Scheduled Castes and government in 1992 passed the 73rd and
Scheduled Tribes. 74th Constitution (Amendments) to lay
Supplementary Notes: the legal and constitutional foundation for
Evolution of the third tier governance in genuine self-governance in India.
India  The 73rd Amendment Act echoed the
 With the exit of the British, the recommendations of the Balwant Rai
institutionalisation of local bodies regressed Mehta Committee. The 73rd Amendment
as several Indian states dissolved district unveiled three-tier rural institutions:
councils, bringing their duties and functions Gram Panchayats at the village level,

RE
within the ambit of state governments, such Panchayat Samitis, and Zilla Parishad
as in Bihar in 1958. at the district level.
 However, the first Five Year Plan (1951-56)
 Further, the act mandated direct and regular
recognised the importance of decentralised
elections every five years and a mandatory
governance, stating that local self-
quota for the SCs and STs.
governing bodies play a vital role in a
democratic system where there is a “proper  Importantly, the Act mandated one-third
diffusion of power and responsibility”. of the seats at all levels of panchayats
 The impetus for establishing local
O are reserved for women.
bodies gained further currency with  Apart from recommending the devolution
recommendations from the Balwant of 29 functions to panchayats, the 73rd
Rai Mehta Committee of 1957. Amendment mandated the formation
SC
 The Committee emphasised that of state finance commissions and
the central function of local bodies provision for grants-in-aid from state
lies in their role in constituting governments to PRIs.
an intermediary between the
general public and the government, L3: Real Challenge
recommending a three-tier structure
of Panchayati Raj (consisting of Zilla
3. Correct option: (b)
Parishads, Panchayati Samitis, and
GS

Gram Panchayats) and reservation for Explanation:


women.
 Statement 2 is incorrect: Under 74th
 While these recommendations were
Amendment Act, the government passed the
accompanied by a rise in the number
landmark Panchayat Extension to Tribal
of panchayats—over 200,000 village
Areas (PESA) Act in 1996 to extend self-
panchayats were functioning in India by
1959—they were largely ineffective due to a
governance to Scheduled and Tribal Areas
O
of the country.
.C
lack of functional and financial autonomy.
 This led to the recommendations of the Supplementary Notes:
S

Ashok Mehta Committee of 1978 which


74th Amendment Act
included direct elections to Panchayats,
TE

mandatory quotas for marginalised  Whereas the 74th Amendment Act made
communities particularly the Scheduled provisions for urban local bodies (ULBs)
Castes and Scheduled Tribes, and “transfer by mandating a three-tier arrangement of
O

(of) substantial quantum of powers from the ULBs, with Nagar Panchayats in peri-urban
State Government to the local bodies”. areas, Municipal Councils in small towns,
FN

 At the same time, the Constitution (64th and the Municipal Corporations (MCs) in
Amendment) Bill and The Constitution (65th bigger cities/metropolis.
Amendment) Bill of 1989 were introduced to
D

 Much like the 73rd Amendment, the 74th


formally institutionalise local bodies at the
Amendment enables direct elections every
.P

national level. The Bills, however, failed to


five years at the municipality level, with a
receive enough support in Parliament.
mandatory quota for SCs and STs apart from
W

L2: Approachable reserving one-third of seats for women.


W

2 PPP-PTS-4273/082023/04
W
https://www.pdfnotes.co/

 The Twelfth Schedule - a list of 18 subjects Supplementary Notes:


that can be devolved to ULBs in due
course—was established. The Act also Panchayat (Extension to Scheduled
provides for the creation of a Finance Areas) Act (or PESA), 1996
Commission to determine the sharing  Village level democracy became a real
of taxes, duties, and funds from the prospect for India in 1992 with the 73rd
State Consolidated Fund. amendment to the Constitution, which
mandated that resources, responsibility
 Lastly, the government passed the and decision making be passed on from
landmark Panchayat Extension to central government to the lowest unit of the
Tribal Areas (PESA) Act in 1996 to governance, the Gram Sabha or the Village
extend self-governance to Scheduled Assembly.
and Tribal Areas of the country.
 A three tier structure of local self-government
 Significantly, the Act reserves 50 percent was envisaged under this amendment.
of local body seats for STs, while Since the laws do not automatically cover
reserving the office of chairperson the scheduled areas, the PESA Act was in
solely for the ST community. In short, acted on 24 December 1996 to enable Tribal
these three legislations have formed the Self Rule in these areas.
key pillars of local self-governance in India,
 The Act extended the provisions of
forming the foundations upon which today’s

RE
Panchayats to the tribal areas of nine states
third-tier institutions rest. that have Fifth Schedule Areas.
L3: Real Challenge  The nine states with Fifth Schedule areas
are: Andhra Pradesh, Chhattisgarh,
Gujarat, Himachal Pradesh, Jharkhand,
4. Correct option: (b) Maharashtra, Madhya Pradesh, Orissa,
Explanation: and Rajasthan.

 Option (b) is correct


Supplementary Notes:
Bhuria Committee
O  The fundamental spirit of the Panchayat
Extension Act for tribal areas under 5th
Schedule is that it devolves power and
authority to Gram Sabha and Panchayats
rather than delegation; hence it paves way
SC
 The Rao government appointed a committee for participatory democracy.
headed by Mr. Dileep Singh Bhuria, MP,  The provision under constitution and the
in June 1994, to work out the details as to composition under this act call for every
how structures similar to Panchayati Raj legislation on the Panchayat in 5th Schedule
Institutions can take shape in Tribal Areas area are in conformity with the customary
and Scheduled Areas and to define their law, social and religious practices and
powers. traditional management practices of the
community resources. The provisions under
 The Committee submitted its report in
GS

PESA include the following:


January 1995. The Bhuria Committee
recommended a three-tier structure of  Gram sabha at the para, majra and tola
self-governance in the tribal areas: levels
 Gram Sabha - Every “habitation  Gram sabha to protect the traditions,
community” to have a Gram Sabha beliefs and culture of the tribal O
which will exercise command over communities
natural resources, resolve disputes and Local disputes to be resolved by the gram
.C

manage institutions under it like schools sabha
and cooperatives
Gram sabha to manage and protect
S


 Gram Panchayat - Elected body of common properties based on their
TE

representatives of each Gram Sabha, traditional systems of management and


also to function as an appellate authority protection
for unresolved disputes at lower level
The administration to seek permission
O


 A block or taluka level body as the next from the gram sabha in case of land
higher level. acquisition
FN

L2: Approachable  Gram sabha to have the rights over


minor forest produce; powers to restore
D

land to the tribals; and control over


5. Correct option: (d) money-lending to tribals, tribal welfare
.P

activities by social organizations and local


Explanation: plans and sub-plans for the development
W

 Option (d) is correct of tribal areas and communities


W

PPP-PTS-4273/082023/04 3
W
https://www.pdfnotes.co/

 Gram sabha to have the control over Supplementary Notes:


local markets and melas
Eleventh Schedule (Article 243G)
 Gram sabha to have rights to control the
distillation, prohibition and manufacture  The 11th Schedule was added to the Indian
of liquor Constitution by the 73rd Amendment
Act of 1992, which is also known as the
 District panchayats to have rights and Constitution (Seventy-Third Amendment)
powers similar to the district panchayats Act.
falling under Sixth Schedule
 This amendment aimed to strengthen
L2: Approachable the Panchayati Raj system in India
by empowering local self-government
institutions at the village, intermediate,
6. Correct option: (a) and district levels.
Explanation:  The Panchayati Raj system was established
 Option (a) is correct to decentralize power and governance,
allowing local communities to participate
Supplementary Notes: actively in the decision-making processes
that affect them.
Urban Local Bodies (ULBs)

RE
 The 11th Schedule outlines the functions
 ULBs are small local bodies that administer
and powers to be entrusted to Panchayats
or govern a city or a town of specified
(village councils) and highlights the areas in
population. Urban Local Bodies are vested
which they can exercise their authority. The
with a long list of functions delegated to them
functions and powers outlined in the 11th
by the state governments. These functions Schedule can be summarized as follows:
broadly relate to public health, welfare,
regulatory functions, public safety, public  Agriculture, including agricultural
infrastructure works, and development
O extension.
activities.  Land improvement, implementation
 There are several types of Urban Local bodies of land reforms, land consolidation
in India such as Municipal Corporation, and soil conservation.
SC
Municipality, Notified Area Committee,  Minor irrigation, water management
Town Area Committee, Special Purpose and watershed development.
Agency, Township, Port Trust, Cantonment
Board etc. The main sources of revenues of  Animal husbandry, dairying and
ULBs are as follows: poultry.

 Collection from tax and non-tax sources  Fisheries.


as assigned to them under Municipal  Social forestry and farm forestry.
act.
GS

 Minor forest produce.


 Devolution of shared taxes and duties
as per recommendation of State Finance  Small scale industries, including food
Commission processing industries.

 Grants-in-aid from the State  Khadi, village and cottage industries. O


Government  Rural housing
.C
 Grants-in-aid from the Government  Drinking water
of India under Centrally Sponsored
Schemes  Fuel and fodder
S

Share of State Govt. of State against  Roads, culverts, bridges, ferries,


TE


Centrally Sponsored Schemes of Govt. of waterways and other means of
India communication.
O

 Award of Central Finance Commission  Rural electrification, including


Grant, GOI distribution of electricity.
FN

L2: Approachable  Non-conventional energy sources.


 Poverty alleviation programme
D

7. Correct option: (c)  Education, including primary and


.P

secondary schools
Explanation:
 Technical training and vocational
W

 Option (c) is correct education


W

4 PPP-PTS-4273/082023/04
W
https://www.pdfnotes.co/

 Adult and non-formal education  Urban forestry, protection of the


environment and promotion of ecological
 Libraries
aspects.
 Cultural activities
 Safeguarding the interests of weaker
 Markets and fairs sections of society, including the
handicapped and mentally retarded.
 Health and sanitation, including
hospitals, primary health centres and  Slum improvement and upgradation.
dispensaries.  Urban poverty alleviation.
 Family welfare.  Provision of urban amenities and facilities
 Women and child development. such as parks, gardens, playgrounds.
 Social welfare, including welfare of the  Promotion of cultural, educational and
handicapped and mentally retarded. aesthetic aspects.

 Welfare of the weaker sections, and in  Burials and burial grounds; cremations,
particular, of the Scheduled Castes and cremation grounds and electric
the Scheduled Tribes. crematoriums.

 Public distribution system.  Cattle pounds; prevention of crue;ty to

RE
animals.
 Maintenance of community assets
 Vital statistics including registration of
L3: Real Challenge births and deaths.
 Public amenities including street
8. Correct option: (b) lighting, parking lots, bus stops and
public conveniences.
Explanation: O  Regulation of slaughter houses and
 Statement 1 is incorrect: The XIIth tanneries.
Schedule was added by the 74th Amendment
Act of 1992 to empower urban local bodies L2: Approachable
and ensure decentralized governance at the
SC
grassroots level.
9. Correct option: (d)
 Statement 2 is correct: It aims to
strengthen local self-government and
Explanation:
promote effective and efficient urban  All statements are correct
planning and development.
Supplementary Notes:
Supplementary Notes:
Local Governance
XIIth Schedule of the Indian
GS

 Local governance in India refers to the


Constitution system of governing at the grassroots
 The XIIth Schedule outlines the level, where local bodies or institutions are
functional domains and responsibilities of responsible for managing and addressing
municipalities and urban local bodies. It the needs and concerns of the people in their
consists of 18 items that cover a wide range respective areas. O
of subjects relating to urban governance.  The two primary tiers of local governance
.C
These items are as follows: in India are the rural local bodies known as
 Urban planning including town Panchayats and the urban local bodies
S

planning. called Municipalities or Municipal


Corporations.
TE

 Planning of land- use and construction


of buildings. Significance of Local Governance in
India:
Planning for economic and social
O


development.  Decentralization of Power:
FN

Local governance ensures the


 Roads and bridges. decentralization of power from the
 Water supply for domestic, industrial central and state governments to the
local bodies. This helps in better and more
D

and commercial purposes.


effective management of resources and
Public health, sanitation conservancy
.P

 decision-making at the local level, leading


and solid waste management. to greater responsiveness to local needs and
W

 Fire services. issues.


W

PPP-PTS-4273/082023/04 5
W
https://www.pdfnotes.co/

 Community Participation: Local Features and functions of the State


governance encourages citizen Election Commission in India:
participation in the decision-making
process. It allows people to actively Constitutional Status
engage with their local representatives and  The State Election Commission derives its
contribute to the development and planning authority from Article 243K and Article
of their communities. 243ZA of the Constitution of India.
 Proximity to the People: Local  These articles provide for the establishment
governments are closer to the people they and composition of the State Election
serve, allowing them to have a better Commission and its powers and functions.
understanding of the local issues and
challenges. This proximity enables quicker Independent Body
and more accurate identification of problems
 The State Election Commission is intended
and implementation of suitable solutions.
to be an independent and autonomous body.
 Service Delivery: Local governance It is not under the control of the state
is responsible for providing various government.
essential services like water supply,
sanitation, education, healthcare,  The Chief State Election Commissioner
infrastructure development, and more. and other Election Commissioners are

RE
Effective local governance ensures efficient appointed by the Governor of the state,
delivery of these services, leading to an and they are usually retired senior civil
improved quality of life for the citizens. servants or judicial officers.

 Accountability and Transparency: Local Conducting Elections


governance fosters a sense of accountability
 The primary responsibility of the State
among elected representatives, as they are
Election Commission is to conduct elections
directly answerable to their constituents.
O
Moreover, transparency in decision-making
to local bodies in the state.
processes helps in reducing corruption and  This includes preparation of electoral rolls,
promoting good governance. delimitation of constituencies, issuance of
Problems Associated with Local notifications for elections, and supervision
SC

Governance in India - Financial of the entire election process.
Constraints, Administrative Capacity, L3: Real Challenge
Political Interference, Inequality and
Disparities, Limited Autonomy, Lack
of Awareness and Participation, and 11. Correct option: (c)
Corruption.
Explanation:
L1 : Elementary
Statement 2 is incorrect: The State
GS


Election Commission has authority to
10. Correct option: (c) enforce the Model Code of Conduct during
the election period.
Explanation:
 Statement 2 is incorrect: It is not under
Supplementary Notes: O
the control of the state government. State Election Commission
.C
Supplementary Notes:  The State Election Commission is
responsible for preparing and updating the
State Election Commission
S

electoral rolls for local body elections. These


 In India, the State Election Commission rolls include the names of eligible voters in
TE

(SEC) plays a vital role in ensuring free each constituency.


and fair elections at the state level. It
is a constitutional body established  The SEC is responsible for the
O

under the provisions of the Indian delimitation of constituencies for local


Constitution. body elections. This involves the division
FN

of a geographical area into various electoral


 The role of the State Election wards, ensuring equitable representation
Commission is primarily focused and population balance.
D

on conducting elections to the local


bodies, such as municipal corporations,  The State Election Commission issues
.P

municipal councils, panchayats, and notifications to announce the schedule of


other urban and rural local bodies elections, the last date for filing nominations,
W

within the state. and the date of polling and counting.


W

6 PPP-PTS-4273/082023/04
W
https://www.pdfnotes.co/

 The SEC enforces the Model Code of  It is an important constitutional body that
Conduct during the election period. helps in the effective functioning of local
This code lays down guidelines for political governance and promotes decentralization
parties and candidates to ensure fair play of financial resources. The main functions
and ethical campaigning. and responsibilities of the State Finance
 The State Election Commission also deals Commission are as follows:
with disputes related to local body elections.
It adjudicates complaints regarding the Distribution of Financial Resources
conduct of elections, electoral malpractices,  One of the primary functions of the SFC is to
and other related issues. recommend the distribution of net proceeds
 The SEC has the power of superintendence, of taxes between the state government and
direction, and control over all aspects of the the local bodies.
conduct of local body elections, ensuring the
 It suggests the share of revenue that
smooth functioning of the electoral process.
should be allocated to the Panchayats and
 The State Election Commission may Municipalities from the state’s consolidated
provide advice to the state government fund.
on matters related to the conduct of
local body elections, electoral reforms,  This distribution is essential to ensure that
and other election-related issues. local bodies have adequate resources to

RE
L2: Approachable carry out their functions effectively.

Determining Grants-in-Aid
12. Correct option: (a)  The SFC also recommends the grants-
in-aid to be given to the Panchayats and
Explanation:
Municipalities from the state government’s
 Both statements are correct revenue.
Supplementary Notes:
Finance Commission
O  These grants are meant to supplement the
financial resources of the local bodies and
bridge any gaps between their revenue
 Our Constitution has made provision for generation capacity and expenditure
SC
the creation of Finance Commission under requirements.
article 280, to recommend devolution of
resources from the Center to the States Assessing Local Body Finances
through Tax-Sharing and Grants-in-
Aid, no such Constitutional arrangement  The SFC examines the financial position
was made for a long time since independence of Panchayats and Municipalities and
for recommending transfer of resources from makes assessments of their income and
the State Government to Local Bodies. expenditure patterns.
GS

 Thus, Local Bodies were handicapped in  This assessment helps in understanding


their functioning and their development the fiscal capacity of the local bodies and
because of a number of inadequacies and formulating appropriate fiscal policies.
limitations – Constitutional, institutional,
functional and financial. Reviewing Financial Devolution O
L2: Approachable  The Commission periodically reviews the
financial devolution to local bodies and
.C
makes recommendations for the equitable
13. Correct option: (a)
sharing of resources.
S

Explanation: It takes into account the changing needs


TE


 Option (a) is correct and responsibilities of local bodies and
ensures that they have adequate financial
Supplementary Notes:
autonomy to carry out their functions.
O

State Finance Commission


Rationalization of Local Taxes and Fees
FN

 The role of the State Finance Commission


(SFC) in India is enshrined in the Indian  The SFC suggests measures for improving
Constitution under Article 243-I and Article the revenue base of Panchayats and
D

243-Y. Municipalities.
.P

 The SFC plays a crucial role in ensuring  It may recommend rationalization of local
fiscal autonomy, equity, and efficiency in taxes, fees, and other sources of revenue to
W

the financial affairs of the states. enhance their financial self-sufficiency.


W

PPP-PTS-4273/082023/04 7
W
https://www.pdfnotes.co/

Fiscal Consolidation and Management  Under clause (2), Parliament is also


entitled to use the state machinery for
 The Commission also advises on measures to
the enforcement of the union laws, and
improve fiscal management and governance
confer powers and entrust duties to
at the local level.
the state.
 It promotes fiscal prudence and efficiency  A state can also, with the consent of union
in the utilization of resources by the government confer administrative functions
Panchayats and Municipalities. on the union.
Reporting to the Governor L3: Real Challenge
 After examining the financial aspects of
local bodies and considering their financial
requirements, the State Finance Commission
15. Correct option: (b)
submits a report to the Governor of the Explanation:
state.
 Statement 1 is incorrect: The provisions
 The report contains its recommendations relating to the financial relations between
on financial matters related to local the union and the states are derived from
governance. the Government of India Act, 1935.
L3: Real Challenge Statement 2 is correct: The seventh

RE

schedule of the Constitution provides for
specific entries reserved for the union and
14. Correct option: (b) the states for imposing taxes.
Explanation: Supplementary Notes:
 Statement 3 is incorrect: The union can Financial relations between Centre and
give directives to the state pertaining to the
states
state.
O
construction and maintenance within the
 The areas of taxation have been clearly
demarcated between the centre and states.
Supplementary Notes: The states have little powers in taxation
Administrative relations between Centre and are heavily dependent on the centre,
SC
and states for financial resources. The chief source of
finance of the states is the grants-in-aid
 The executive power of the union from the centre. ‘The seventh schedule of
also extends to giving of direction to the Constitution provides for specific entries
the state under Article 256 for their reserved for the union and the states for
compliance. imposing taxes. The union can levy taxes on
the 12 items of Union List (82 to 92 A).
 This power of the Union extends to the limit
of directing a state in a manner it feels  Similarly, the state list contains 19 items on
GS

essential for the purpose. which states are empowered to conect taxes.
The residuary powers in taxation vests with
 For instance, the union can give directives to
Parliament.
the state pertaining to the construction and
maintenance of means of communication  There is a four-fold classification of tax
declared to be of national or military revenues between the union and the states. O
importance or protection of railways within These are:
the state.
Taxes levied by the union but coIlected and
.C

 This is essential to ensure the whoIly appropriated by the state (Article
implementation of parliamentary laws 270). These are stamp duties and duties of
S

throughout the country. excise on medicinal and toilet preparations.


TE

 Non-compliance of the directives might lead  Taxes levied and collected by the centre,
to a situation where the union can invoke but wholly assigned to the states (Article
Article 356, for imposition of President’s rule 269). These include duties on succession
O

in the state and take over the administration to property other than agricultural
of state. land, estate duty on property other than
FN

agricultural land, terminal taxes on goods


 Under the constitutional provision of and passengers (railway, sea or air), taxes
Article 254 the President may, with the on railway fares and freights etc.
D

consent of the state government entrust


either conditionally or unconditionally to  Taxes levied and collected by tile union
.P

the government, functions relating to any and distributed between the union and the
matter falling within the ambit of union states (Article 270). This includes taxes on
W

executive power. income other than agricultural income.


W

8 PPP-PTS-4273/082023/04
W
https://www.pdfnotes.co/

 Taxes levied and collected by the union but  The audit reports of the Comptroller
may be shared with the states. This includes and Auditor General are placed before
the customer and excise duties if parliament Parliament or the legislature of the State or
by law so provides. the Union Territory, as the case may be.
L3: Real Challenge  The duties of the Comptroller and Auditor
General also extend to audit of Government
companies and corporations and bodies and
16. Correct Option: (b) authorities in accordance with the laws
made by the legislature and rules made
Explanation:
there under.
 Statement 1 is correct: The Comptroller
L2: Approachable
and Auditor General is also responsible
of compilation of accounts of the State
Government, authorisation of pensionary 17. Correct Option: (c)
benefits of selected categories of employees,
maintenance of provident funds accounts of Explanation:
State Government employees of most State  Statement 1 is incorrect: He can be
Governments. removed by the President at any time. He
can quit by submitting his resignation only

RE
 These audit and accounting functions are
carried out by the Indian Audit and Accounts to the President. Since he is appointed by
the President on the advice of the Council
Department (IA & AD) which functions
of Ministers, conventionally he is removed
under the Comptroller and Auditor General
when the council is dissolved or replaced.
of India.
 Statement 2 is correct: The First Attorney
 Statement 2 is correct: The Comptroller
General of India, M.C. Setalvad, held the
and Auditor General (CAG) is a
position for the longest time—13 years—
O
constitutional functionary, independent of
Parliament/legislature and executives. The
CAG is responsible for;
while Soli Sorabjee held the position for the
shortest time. Nevertheless, he was twice
appointed to the position.
 Audit of Ministries and departments Statement 3 is incorrect: The Office of
SC

of Government of India and the State Profit is used in Article 102 (1)(a) of the
Governments. Indian Constitution which bars a member
 Audit of Central and State Government of the Indian Parliament from holding
Public Sector Undertakings and other an office that would give its occupant the
autonomous bodies and authorities opportunity to gain a financial advantage
which are financed from Government or benefit. It refers to a post under central/
funds. state government which yields salaries,
perks and other benefits.
GS

 Audit of receipts of Union or of States.


 There are no such provisions for
 Audit of accounts stores and stock. Attorney general of India.
 Audit of companies and corporations.  Statement 4 is incorrect: The Law
 Statement 3 is incorrect: CAG can Commission of India is a non-statutory body
O
audit up to an extent which an activity, constituted by the Government of India
from time to time.
programme or organisation operates
.C
economically, efficiently and effectively  The Law Commission undertakes research
known as performance audit for all the in law and review of existing laws in India
S

eligible audits done by him. for making reforms therein and enacting
TE

new legislations on a reference made to it


Supplementary Notes: by the Central Government or suo-motu.
Role of the C&AG in regard to audit: Law and Legislative Secretaries in
O


 The Comptroller and Auditor General is the the Law Ministry will be the ex-officio
members of the commission and not the
FN

sole authority prescribed in the Constitution


entrusted with the responsibility of audit of Attorney general of India.
accounts of the Union and of the States.  It will also have not more than five part-
D

time members.
 It is the duty of the Comptroller and Auditor
.P

General to audit receipts and expenditure  A retired Supreme Court judge or Chief
of the Union and each State and the Union Justice of a High Court will head the
W

Territory Governments. Commission.


W

PPP-PTS-4273/082023/04 9
W
https://www.pdfnotes.co/

Supplementary Notes: Supplementary Notes:


Attorney General of India: Union Public service commission
(UPSC)
 The attorney general is chosen by the
President of India. He must be an Indian  The Full Form of UPSC is Union Public
citizen and have completed 10 years of high Service Commission. It was established
court advocacy or 5 years as a judge in any on October 1, 1926, under the name
Public Service Commission. After being
Indian state.
reconstructed in 1935 under the Government
 He may also be regarded as a prominent of India Act 1935, the Commission’s name
jurist by the President. The duration of was changed to Federal Public Service
the Attorney General’s appointment is not Commission.
specified in the Constitution.  It is an important institution in India that
 The Constitution is also silent regarding the plays a significant role in the country’s
governance and administration.
removal procedure and the justifications for
removal. As a result, they are subject to the  It is responsible for conducting prestigious
president’s whim and are thus subject to civil service examinations and selecting
removal at any time. candidates for various positions in the

RE
government.
 He or she can also step down by giving the
President a letter of resignation.  The UPSC ensures a fair and merit-
based selection process, aiming to recruit
 There are no constitutional limitations on individuals with the necessary skills and
the Attorney General’s remuneration, and it knowledge to serve the nation effectively.
is determined at the President’s discretion. L2: Approachable
L2: Approachable
O
19. Correct Option: (d)
18. Correct Option: (a) Explanation:
Explanation: All statements are correct.
SC

 Statement 1 is correct: Under Article Supplementary Notes:
320 of the Constitution of India, the
Joint Public service Commission (JPSC)
Commission is, inter-alia, required to
be consulted on all matters relating to  The Union Public Service Commission
(UPSC) is the central recruiting agency in
recruitment to civil services and posts.
India.
 The functions of the Commission under  It is an independent constitutional body.
GS

Article 320 of the Constitution are:


 The provisions regarding the composition
 Conduct examinations for appointment of UPSC, appointment and removal of its
to the services of the Union. members and the powers and functions
of UPSC are provided in Part XIV of the
 Direct recruitment by selection through Indian Constitution under Article 315 to
interviews. Article 323.
O
 Appointment of officers on promotion /  Parallel to the UPSC at the Centre, there is
.C
deputation / absorption. a State Public Service Commission (SPSC)
in the state.
S

 Framing and amendment of Recruitment


 The provisions regarding the composition
Rules for various services and posts of SPSC, appointment and removal of its
TE

under the Government. members and the powers and functions


 Disciplinary cases relating to different of SPSC are provided in Part XIV of the
Indian Constitution under Article 315
O

Civil Services.
to Article 323.
FN

 Advising the Government on any matter  A JSPSC is a statutory body. It is not a


referred to the Commission by the constitutional body. After the creation of
President of India. Haryana out of Punjab in 1966, the two
D

states of Punjab and Haryana had a JSPSC


 Statement 2 is incorrect: The formation
for a short period.
.P

of a parmanent commission for civil services


was bought from the Government of India  The President appoints the chairman as
W

Act 1935. well as the members of a JSPSC.


W

10 PPP-PTS-4273/082023/04
W
https://www.pdfnotes.co/

 In 1926 on the recommendation of Lee for, managing some or all of the elements
Commission (1924), the Federal Public that are essential for the conduct of elections
Service Commission was established. and direct democracy instruments—such
as referendums, citizens’ initiatives and
 Government of India Act, 1935 provided for
recall votes—if those are part of the legal
Central Public Service Commissions, State
framework. These essential (or core)
Public Service Commission and Joint Public
elements include:
Service Commission.
 determining who is eligible to vote;
 The Constitution makes provision for a JPSC
to cater to needs of two or more states.  receiving and validating the nominations
of electoral participants (for elections,
 JPSC is formed by an act of parliament.
political parties and/or candidates);
Thus, a JSPSC is a statutory and not a
constitutional body.  conducting polling;
 It submits reports to the concerned State  counting the votes; and
Governors.
 Tabulating the votes.
 President decides composition, tenure,
conditions of service, removal. Chairman  If these essential elements are allocated to
and Members have a term of six years or till various bodies, then all bodies that share
62, whichever is earlier. these responsibilities can be considered

RE
 They can be suspended or removed by the EMBs. An EMB may be a stand- alone
president, or they can resign by writing to institution, or a distinct management unit
the president. within a larger institution that may also
have non-electoral tasks.
 Constitutional Provisions:
 In addition to these essential elements,
 Article 315: Constitution of Public
an EMB may undertake other tasks that
Service Commissions (PSC) for the
assist in the conduct of elections and


Union and for the States of India.
O
Article 316: Appointment and term of
office of members of UPSC as well as
SPSC.
direct democracy instruments, such as
voter registration, boundary delimitation,
voter education and information, media
monitoring and electoral dispute resolution.
SC
 Article 317: Removal and suspension of However, a body that has no electoral
a member of both the UPSC and SPSC. responsibilities other than, for example,
 Article 318: Power to make regulations boundary delimitation (such as a boundary
for the conditions of service of members delimitation commission), electoral dispute
and staff of the Commission. resolution (such as an electoral court),
election media monitoring (such as a media
 Article 319: Prohibition of holding the monitoring commission), or the conduct of
office by members of Commission upon voter education and information (such as a
ceasing to be such members.
GS

civic education commission) is not considered


 Article 320: States the functions of Public an EMB because it is not managing any of
Service Commissions. the essential elements identified above.
 Article 321: Power to extend the functions  Similarly, a national population or statistics
of Public Service Commissions. bureau that produces electoral registers as
part of the general process of population
 Article 322: Expenses of Public Service
registration is not considered to be an
O
Commissions.
.C
 Article 323: Reports of Public Service EMB.
Commissions.  Different EMBs may be established for
S

L2: Approachable different electoral processes. In Mexico and


Poland, the EMBs are responsible for both
TE

presidential and parliamentary elections;


20. Correct Option: (c) in Australia, the national EMB deals with
national-level elections, while state-level
O

Explanation:
elections are the responsibility of separate
FN

 All statements are correct. state-level EMBs. In the United Kingdom


(UK), the arrangements for the conduct of
Supplementary Notes:
elections and referendums are separate.
D

Election Commission as an Electoral  Some bodies that are not engaged in any
management Body of the essential elements of elections may
.P

 An EMB is an organization or body that has nonetheless be popularly regarded as


EMBs.
W

the sole purpose of, and is legally responsible


W

PPP-PTS-4273/082023/04 11
W
https://www.pdfnotes.co/

 The US Federal Election Commission (FEC) devolved to states has constituted over
defines its mission as ‘administering and 80% of the total central transfers to states
enforcing federal campaign finance laws’. (Figure 1). The centre also provides grants
However, such institutions do not qualify as to states and local bodies which must be
EMBs under the definition above. used for specified purposes. These grants
have ranged between 12% to 19% of the
 In addition to the division of functional
total transfers.
responsibility for different elements of the
electoral process, electoral responsibilities L2: Approachable
may be divided between bodies at different
levels.
22. Correct Option: (c)
 For example, some elements of the conduct
of elections may be managed by a national- Explanation:
level electoral commission, a ministry (such  Statement 1 is correct: The GST Council,
as the Ministry of the Interior) or a national consisting of the Union Finance Minister
government agency, while others are and representatives from all States and
implemented by local-level commissions, Union Territories.
regional branches of government
departments or local authorities (as in  Statement 2 is correct: It was established
Spain). to make decisions on various aspects of

RE
GST, including tax rates, exemptions, and
L3: Real Challenge administrative procedures. It played a
crucial role in shaping the GST framework
in India.
21. Correct Option: (b)
 Statement 3 is correct: The GST Council
Explanation: was notified w.e.f. 12th September, 2016.
Statement 1 is incorrect: The Finance For assisting the GST Council, the office of

O
commission can recommend the distribution
the GST Council Secretariat was also
established.
of rural local bodies’ grants to all three
tier of government which includes; Gram  GST Council Secretariat is responsible for
Panchayats, Block panchayats and conducting meetings and setting up the
SC
District panchayats. agenda for those meetings.
 Statement 2 is correct: Besides the taxes  Statement 4 is incorrect: Union Revenue
devolved to states, another source of transfers Secretary is the ex-officio Secretary to the
from the centre to states are grants-in- GST Council.
aid. As per the recommendations of the
Supplementary Notes:
14th Finance Commission, grants-in-aid
constitute 12% of the central transfers GST Council and Working
to states.
GS

 On July 1, 2017, GST laws were


 The 14th Finance Commission had implemented, replacing a complex web of
recommended grants to states for three Central and State taxes. Under the Indian
purposes: (i) disaster relief, (ii) local GST, goods and services are categorized
bodies, and (iii) revenue deficit. into different tax slabs, including 5%, 12%,
18%, and 28%.
Supplementary Notes:
 Some essential commodities are exempted
O
Finance commission from GST, Gold and job work for diamond
.C
 The Finance Commission is a constitutional attract low rate of taxation. Compensation
body formed every five years to give cess is being levied on demerit goods and
S

suggestions on centre-state financial ceratin luxury items.


TE

relations. Each Finance Commission is  To prepare for the implementation of GST,


required to make recommendations on: (i) extensive efforts were made to build the
sharing of central taxes with states, (ii) necessary technological infrastructure
O

distribution of central grants to states, (iii) and train tax officials and businesses.
measures to improve the finances of states GST Network (GSTN), a not-for-profit
FN

to supplement the resources of panchayats company, was created to provide the IT


and municipalities, and (iv) any other backbone for the GST system, including
matter referred to it. taxpayer registration, return filing, and tax
D

payments.
 Composition of transfers: The central
.P

taxes devolved to states are untied funds,  Since its implementation, the Indian
and states can spend them according GST has undergone various amendments
W

to their discretion. Over the years, tax and refinements based on feedback from
W

12 PPP-PTS-4273/082023/04
W
https://www.pdfnotes.co/

businesses and the evolving economic 24. Correct Option: (c)


scenario. While the GST implementation
initially posed challenges for businesses in Explanation:
terms of understanding the new compliance  Statement 1 is correct: The Committee
requirements and adapting to the changes, on the Status of Women in India (CSWI)
it has gradually settled into the Indian tax recommended nearly two decades ago, the
landscape. setting up of a National Commission for
 It can be said that the history of GST in women to fulfil the surveillance functions
India showcases a monumental shift in the to facilitate redressal of grievances and to
country’s tax structure, aiming to create accelerate the socio-economic development
a more unified, efficient, and transparent of women.
indirect tax regime for the benefit of  Successive Committees/Commissions/Plans
businesses and the economy as a whole. including the National Perspective Plan
for Women (1988-2000) recommended the
L3: Real Challenge
constitution of an apex body for women.
 Statement 2 is correct: The Commission
23. Correct Option: (c) being a Coordinating Agency at the
Explanation: National Level to deal with issues

RE
pertaining to NRI marriages formally
 Statement 1 is incorrect: The National
created NRI Cell in 2009.
Commission for Scheduled castes
(NCSC) is a Constitutional body  The NRI Cell deals with complaints and
established with a view to provide safeguards specific issues related to the NRI marital
against the exploitation of scheduled disputes causing deprivation of women’s
castes and Anglo Indian communities rights, such as Indian women who are
to promote and protect their social, aggrieved of desertion by NRI/Overseas
educational, economic and cultural
O
interests, specials provisions were made in
the constitution. 
husband in India, abandonment in foreign
land or any other related issues
Statement 3 is correct: NCW under
Statement 2 is incorrect: Atrocities Section 10(1) (f) of the National
SC

and Protection of civil rights wing Commission for Women Act, 1990 takes
of the NCSC protects the cases relating suo motu cognizance of matters where there
to protection of civil rights act, the is:
minimum wages act etc.  Violation /deprivation of the rights of
Supplementary Notes: women.
 Non-implementation of laws enacted to
National Commission for Scheduled
provide protection to women and also to
GS

Castes (NCSC) achieve the objective of equality.


 The National Commission for Scheduled  Non-compliance of policy decisions,
Castes is an Indian constitutional body guidelines or instructions aimed at
under the jurisdiction of Ministry of Social mitigating hardships and ensuring
Justice and Empowerment, Government welfare and empowerment of women. O
of India established with a view to provide
safeguards against the exploitation of  Suo-Motu cognizance may be taken on
.C
Scheduled Castes and Anglo Indian the basis of print, electronic and social
communities to promote and protect their media or any other information received
S

rights. by the Commission.


TE

 Statement 4 is correct: The Legal Cell of the


Commission reviews the constitutional and
legal safeguards provided, recommending
O

remedial legislative measures and facilitates


redressal & offers advice to Government.
FN

Supplementary Notes:
National commission for Women (NCW)
D

 The National Commission for Women was set


.P

up as statutory body in January 1992 under


the National Commission for Women
L2: Approachable
W

Act, 1990 to review the Constitutional and


W

PPP-PTS-4273/082023/04 13
W
https://www.pdfnotes.co/

legal safeguards for women; recommend and Zoroastrians (Parsis) were notified
remedial legislative measures, facilitate as minority communities by the Union
redressal of grievances and advise the Government. Further, Jains were
Government on all policy matters affecting also notified as another minority
women. community.
 Functions of Women Safety Cell: Supplementary Notes:
 Work relating to Project – Violence free National Commission for Minorities
homes set up in Delhi and seven other
States including monitoring the working  The Ministry of Minority Affairs was
of special cells, release of funds for project carved out of Ministry of Social Justice
execution and institutionalization and & Empowerment and created on 29th
takeover of the project by the State January, 2006 to ensure a more focused
approach towards issues relating to the
 Monitoring of cases of acid attack notified minority communities namely
and matters relating to payment of Muslim, Christian, Budhist, Sikhs, Parsis
compensation in such cases in all and Jain.
States and UTs and providing inputs
for formulation of Victim Compensation  The mandate of the Ministry includes
Schemes. formulation of overall policy and planning,
coordination, evaluation and review of the

RE
 MIS for Acid Attack Victims regulatory framework and development
programmes for the benefit of the minority
 Women Welfare and Capacity Building
communities.
Cell:
 Vision: Empowering the minority
 Formally setup in 2019, WW&CB
communities and creating an enabling
Cell works on the needs of women-
environment for strengthening the multi-
centric interventions to address the
racial, multi-ethnic, multi-cultural, multi-
most pressing challenges affecting
O lingual and multi-religious character of our
empowering of women at economic,
nation.
social, cultural and political level.
 Mission: To improve the socio-economic
 The Cell has been conducting various
conditions of the minority communities
SC
Awareness programs, Workshops,
through affirmative action and inclusive
Conferences, Seminars, Competitions,
development so that every citizen has
Legal Awareness Programmes, Gender
equal opportunity to participate actively in
sensitization workshops, capacity
building a vibrant nation. To facilitate an
building, women empowerment and
equitable share for minority communities in
personality developments programs, etc.
education, employment, economic activities
for holistic growth of women, individually
and to ensure their upliftment.
and also in collaboration with multiple
L2: Approachable
GS

stakeholders.
L3: Real Challenge
26. Correct option: (a)
25. Correct Option: (c)  Statement 1 is incorrect: The President
can increase or decrease the area of any O
Explanation: Scheduled Area in a State.
.C
 Statement 1 is correct: Union  Statement 2 is correct: The Governor may
Government constituted National make regulations for the peace and good
Commission for Minorities, New Delhi government of any area in a State which is
S

and State Government constituted State for the time being a Scheduled Area.
TE

Minorities Commissions in their respective


State Capitals. These organisations are set-  Statement 3 is incorrect: The Governor is
up to safeguard and protect the interests of empowered to direct that any particular act
of Parliament or the state legislature does
O

minorities as provided in the Constitution


of India and laws enacted by the Parliament not apply to a scheduled area or apply with
FN

and the State Legislatures. specified modifications and exceptions.

 Statement 2 is incorrect: The Union Supplementary Notes:


Government set up the National Commission
D

Administration of Scheduled Areas


for Minorities (NCM) under the National
.P

Commission for Minorities Act, 1992.  The President is empowered to declare


Initially five religious communities, viz., an area to be a scheduled area. He can
W

Muslims, Christians, Sikhs, Buddhists also increase or decrease its area,


W

14 PPP-PTS-4273/082023/04
W
https://www.pdfnotes.co/

alter its boundary lines, rescind such Council is mandatory to provide


designation or make fresh orders guidance on the welfare and progress
for such re-designation on an area in of the scheduled tribes.
consultation with the governor of the
state concerned.  This council will comprise 20 members,
with three- fourths of the seats reserved for
 The scheduled areas in the country are representatives of the scheduled tribes in
distinct from other regions due to their the state legislative assembly.
population of “aboriginals,” who face social
and economic disadvantages.  Additionally, in states with scheduled
tribes but no scheduled areas, the
 Consequently, it is crucial to make special President may direct the formation of
efforts to improve their condition. This a similar council.
leads to a situation where the normal
administrative machinery of a state does  There shall be established in each State
not fully extend to the scheduled areas, and having Scheduled Areas therein and, if the
the Central government assumes greater President so directs, also in any State having
responsibility for these regions. Scheduled Tribes but not Scheduled Areas
therein, a Tribes Advisory Council consisting
 When it comes to executive power, a state
of not more than twenty members of whom,
has jurisdiction over the scheduled areas.
as nearly as may be, three-fourths shall be

RE
However, the Governor bears specific
responsibility for these areas. the representatives of the Scheduled Tribes
in the Legislative Assembly of the State:
 It is mandatory for the governor to submit
an annual report on the administration of  It shall be the duty of the Tribes
the scheduled areas or provide it whenever Advisory Council to advice on such
requested by the president. matters pertaining to the welfare and
advancement of the Scheduled Tribes in
 Furthermore, the Central government the State as may be referred to them by

of these areas.
O
possesses the authority to issue directives
to the states regarding the administration

the Governor.
Tribes Advisory Councils (TAC) has been
constituted in Scheduled Area States of
 The Governor, in consultation with the
Andhra Pradesh, Chhattisgarh, Gujarat,
SC
Tribes Advisory Council, holds the power to
Jharkhand, Himachal Pradesh, Madhya
formulate regulations aimed at maintaining
Pradesh, Maharashtra, Odisha, Rajasthan
peace and promoting good governance in the
and Telangana.
scheduled areas.
 Though TamilNadu, Uttarakhand and West
 These regulations may include provisions
to restrict or prohibit land transfers among Bengal do not have any scheduled area, they
scheduled tribe members, regulate land havealso constituted TAC.
allotment to them, and govern money- L2: Approachable
GS

lending activities related to the scheduled


tribes.
28. Correct option: (c)
 Additionally, these regulations may repeal
or amend any relevant acts of Parliament Explanation:
or state legislature that apply to scheduled O
areas.  Both statements are correct
.C
 However, the assent of the president is Supplementary Notes:
required for all such regulations to be
Administration of Tribal Areas
S

enacted.
 Tribal areas are governed as autonomous
L2: Approachable
TE

districts but remain under the executive


authority of the state.
27. Correct option: (b)
O

 The state’s Governor has the power to


Explanation: organize and reorganize the autonomous
FN

district, including modifying its area and


 Option (b) is correct boundary lines.
Supplementary Notes: If multiple tribes exist within an
D


autonomous district, the Governor is
Scheduled Tribes Advisory Council
.P

authorized to divide it into separate


 In states with scheduled areas, the autonomous regions.
W

establishment of a Tribes Advisory L2: Approachable


W

PPP-PTS-4273/082023/04 15
W
https://www.pdfnotes.co/

29. Correct option: (d) 31. Correct option: (b)


Explanation Explanation:
 Option (d) is correct  Both statements are correct
Supplementary Notes: Supplementary Notes:
Criteria for Declaring Schedule Areas Administration in Tribal Area
 The criteria followed for declaring an area  Within their territorial jurisdictions,
as Scheduled Area are: the district and regional councils
 Preponderance of tribal population possess the authority to establish
village councils or courts dedicated to
 Compactness and reasonable size of the resolving disputes among the tribes.
area
 These councils hear appeals arising from
 Under-developed nature of the area such cases. The Governor specifies the
 Marked disparity in economic standard jurisdiction of the high court over
of the people these suits and cases.

 These criteria are not spelt out in the  The district council is empowered to

RE
Constitution of India but have become establish, construct, or manage essential
well established. They embody principles institutions and facilities within the district,
followed in declaring ‘Excluded’ and such as primary schools, dispensaries,
‘Partially-Excluded’ Areas under the markets, ferries, fisheries, and roads.
Government of India Act 1935, Schedule  It can also formulate regulations governing
B of recommendations of the Excluded and money lending and trading activities
Partially Excluded Areas Sub Committee of conducted by non-tribals. However, the
Constituent Assembly and the Scheduled
O
Areas and Scheduled Tribes Commission
assent of the governor is required for the
implementation of such regulations.
1961.
 Furthermore, the district and regional
L1 : Elementary
councils are authorized to assess and collect
SC
land revenue, as well as impose certain
30. Correct option: (b) specified taxes withintheir jurisdictions.

Explanation  In the autonomous districts and autonomous


regions, the acts passed by the Parliament
 Statement 2 is incorrect: Tribal or the state legislature do not automatically
habitations in few states like Kerala, Tamil apply. Instead, they may be applied with
Nadu, Karnataka and West Bengal are not modifications and exceptions as determined
covered under the fifth schedule. by the relevant authorities.
GS

Supplementary Notes: L2: Approachable


5th Schedule Areas
 The administration and governance of 32. Correct option: (c)
Scheduled Areas, as well as the welfare of Explanation:
O
Scheduled Tribes residing in any State other
than Assam, Meghalaya, Tripura, and Option (c) is correct: The Constitution,
.C

Mizoram, are addressed in the Fifth under Sixth Schedule, contains special
Schedule of the Constitution. provisions for the administration of tribal
S

areas in the four north-eastern states


 However, the Tribal settlements in
of Assam, Meghalaya, Tripura and
TE

Kerala, Tamil Nadu, Karnataka, West


Mizoram.
Bengal, Uttar Pradesh, and Jammu
& Kashmir are not covered under the Supplementary notes:
O

Fifth or Sixth Schedule.


Administrative features of Sixth Schedule
FN

 Currently, there are ten states - Andhra Areas


Pradesh, Chhattisgarh, Gujarat, Himachal
Pradesh, Jharkhand, Madhya Pradesh,  Autonomy within state executive
D

Maharashtra, Odisha, Rajasthan, and authority: The tribal areas in Assam,


Telangana—that have areas governed by Meghalaya, Tripura, and Mizoram have
.P

the provisions of the Fifth Schedule. been established as autonomous districts.


However, these districts remain under the
L2: Approachable
W

executive authority ofthe respective states.


W

16 PPP-PTS-4273/082023/04
W
https://www.pdfnotes.co/

 Governor’s power to organize and  The document is an effort by the government


reorganize: The governor is granted the to regulate movement to certain areas
authority to organize and reorganize the located near the international border of
autonomous districts. India.
 This includes the ability to modify their  This is an offshoot of the Bengal Eastern
geographical boundaries, increase or Frontier Regulations, 1873, which protected
decrease their areas, rename them, and Crown’s interest in the tea, oil and elephant
undertake similar actions. trade by prohibiting “British subjects”
 Division of autonomous districts into from entering into these “Protected Areas”
regions: In the case of multiple tribes (to prevent them from establishing any
residing within an autonomous district, the commercial venture that could rival the
governor has the powerto divide the district Crown’s agents).
into several autonomous regions.
 The word “British subjects” was replaced
 This allows for more specific governance by Citizen of India in 1950. Despite the
tailored to the needs of distinct tribal fact that the ILP was originally created by
communities. the British to safeguard their commercial
 District Council: Each autonomous district interests, it continues to be used in India,
is equipped with a district council comprising officially to protect tribal cultures in

RE
30 members. Among these members, four northeastern India.
are nominated by the governor, while the  There are different kinds of ILP’s, one for
remaining 26 are elected through adult tourists and others for people who intend
franchise. to stay for long-term periods, often for
 The elected members serve a term of employment purposes.
five years, unless the council is dissolved
 ILP’s valid for tourism purposes are granted
earlier. Nominated members hold office


at the governor’s discretion.
Regional Council for each autonomous
region: Additionally, each autonomous
O 
as a matter of routine.
The states which require the permit
are Arunachal Pradesh, Mizoram, and
region within the district has a separate Nagaland.
SC
regional council. L2: Approachable
 The specific functions and composition
of these regional councils may vary, 34. Correct option: (a)
providing local governance at a more
granular level. Explanation:
 These provisions in the Sixth Schedule  Statement 2 is incorrect: However, after
aim to grant autonomy and self- the merger of Plan and Non-Plan schemes,
GS

governance to the tribal areas, while the TSP under went a name change and
still maintaining a connection with the was rebranded as the Scheduled Tribe
state administration. Component (STC) by the Ministry of
L2: Approachable Finance.
Supplementary Notes:
O
33. Correct option: (d) Tribal Sub-Plan (TSP)
.C
Explanation:  TSP was established in 1974-75 as
S

 Statement 1 is incorrect: The states a strategic initiative to promote the


which require the permit are Arunachal development of areas with a significant
TE

Pradesh, Mizoram, and Nagaland. tribal concentration.


Supplementary notes: Identification and Allocation:
O

Inner Line Permit (ILP)  A total of 41 Central Ministries/Departments


FN

have been designated for the earmarking of


 ILP is an official travel document issued by
funds under the STC.
the Government of India to allow inward
travel of an Indian citizen into a protected State Governments are also required to
D


area for a limited period. allocate TSP funds in proportion to the
.P

 It is obligatory for Indian citizens from Scheduled Tribe (ST) population, as per the
outside those states to obtain a permit 2011 Census, in relation to the overall State
W

for entering into the protected state. Plan.


W

PPP-PTS-4273/082023/04 17
W
https://www.pdfnotes.co/

Monitoring Responsibilities:  The primary objectives of the Council were


to address the economic, educational, and
 The monitoring of the TSP plan was
linguistic aspirations of the Bodos, as well as
initially carried out by the former Planning
to safeguard their land rights, socio-cultural
Commission until 2017-18.
heritage, and ethnic identity. Additionally,
 However, starting from the fiscal year 2018- the Council aimed to expedite infrastructure
19, the Ministry of Tribal Affairs took over development inthe BTC area.
the monitoring of the STC plan.
 The Council’s primary focus is to drive
Fund Allocation and Programs: accelerated progress in one of the most
 The Government allocates funds for various underdeveloped regions of Assam,
schemes and programs across Ministries/ with particular attention given to the
Departments to provide targeted financial advancement of the Bodo community in
and physical benefits to the Scheduled areas such as education, preservation of
Tribes. land rights, linguistic aspirations, cultural
heritage, and ethnic identity.
 In 2018, the Ministry of Tribal Affairs
launched an online portal known as the  Moreover, the BTC places significant
Scheduled Tribe Component Management importance on enhancing the economic
Information System (STCMIS). This portal infrastructure throughout the entire area,

RE
facilitates the monitoring of funds allocated with the goal of uplifting marginalized
and released for the welfare of Scheduled communities regardless of their caste,
Tribes in the Union Budget. creed, or religion, who reside in this part of
the country.
Objective of the Scheduled Tribe
Component: L3: Real Challenge
 The main objective of the Scheduled Tribe
Component is to channelize and monitor 36. Correct option: (c)
O
the allocation of resources and benefits
from general sectors within the Central Explanation:
Ministries/Departments.  All statements are correct
 The aim is to ensure the development of
Supplementary Notes:
SC
Scheduled Tribes, at least in proportion
to their population, by directing the flow of Development Council
outlays and benefits towards their welfare.
 To uplift and sustain the socio-economic
L2: Approachable conditions of different backward
communities, the Government of Assam has
35. Correct option: (b) established 33 DevelopmentCouncils.
 These Councils aim to decentralize power
Explanation:
GS

and responsibility, enabling them to address


 Statement 1 is incorrect: In 2003, the specific development needs of their
the Bodoland Territorial Council was respective communities.
established under the Sixth Schedule to the
Constitution of India.  The primary function of these Council
bodies is to design and implement various O
Supplementary Notes: developmental schemes and projects in a
prioritized and need- based manner.
.C
Bodoland Territorial Council
 Each Development Council was created
 The Councle came after the signing of
through separate Gazetted Notifications,
S

the Memorandum of Settlement on 10th


February, 2003, involving the Government with the initial establishments occurring
TE

of India, the Government of Assam, and the in May and June of 2010, and in January of
Bodo Liberation Tigers. 2011.
Additionally, during the period of 2015-
O

 The Bodoland Territorial Council Legislative 


Assembly consists of Speaker, Deputy 2016, a few more Development Councils
FN

Speaker and other elected MCLAs of the were created to promote the socio-
Bodoland Territorial Council. economic, educational, cultural, and ethnic
advancement of specific communities within
D

 The executive and legislative powers of the the state of Assam. Some of the Development
Bodoland Territorial Council are derived councils aíe as given below:
.P

from the provisions of the Sixth Schedule of


the Constitution of India and the 2003 and  Moían Development Council
W

2020 Bodoland Peace Agreements.  Mottok Development Council


W

18 PPP-PTS-4273/082023/04
W
https://www.pdfnotes.co/

 Maimal Development Council given in the Concurrent list, but the


Centre has a prior and supreme claim
 Moíia Development Council
to legislate on current subjects.
 Goíkha Development Council
 In case of conflict between the law of
L3: Real Challenge the State and Union law on a subject
in the Concurrent list, the law of the
Parliament prevails.
37. Correct option: (b)
L2: Approachable
Explanation:
 Both statements are correct
38. Correct Option: (c)
Supplementary Notes:
Explanation:
Legislative relations between Centre and
states  Both statements are correct

 Articles 245 to 255 in Part XI of the Supplementary Notes:


Constitution deal with the legislative
Inter-state councils
relations between the Centre and the
State.  As part of the process of reviewing the

RE
working of the existing arrangements
Extent of laws made by Parliament and
between the Union and the States, the
by the Legislatures of States
Government constituted a commission in
 The Parliament can make laws for the whole 1988 under the Chairmanship of Justice
or any part of the territory of India. Territory R.S. Sarkaria.
of India includes the states, UTs and any
other area for the time being included in  One of the important recommendations
the territory of India. Whereas, the state
O of Sarkaria Commission was for
legislature can make laws for whole or any establishing a permanent Inter-State
part of state. Council as an independent national
forum for consultation with a mandate
 The Parliament can alone make ‘extra well defined in accordance with Article
territorial legislation’ thus the laws of the 263 of the Constitution of India.
SC
Parliament are applicable to the Indian
citizens and their property in any part of  The inter-state council is a
the world. recommendatory body that has been
empowered to investigate and discuss
Subject-matter of laws made by subjects of common interest between
Parliament and by the Legislation of the Union and state(s), or among states.
States It also makes recommendations for better
 The Constitution divides legislative coordination of policy and action on these
authority between the Union and the States subjects, and deliberations on matters of
GS

in three lists- the Union List, the State List general interest to the states, which may be
and the Concurrent List. referred to it by its chairman.
 The Union list consists of 99 items. The  It also deliberates on other matters of
Union Parliament has exclusive authority general interest to the states as may be
to frame laws on subjects enumerated in referred by the chairman to the council. The
O
the list. Council may meet at least thrice in a year.
These include foreign affairs, defence, There is also a Standing Committee of the
.C

armed forces, communications, posts and Council.
telegraph, foreign trade etc. L2: Approachable
S

 The State list consists of 61 subjects on


TE

which ordinarily the States alone can make


laws. These include public order, police, 39. Correct option: (d)
administration of justice, prison, local Explanation:
O

governments, agriculture etc.


 All pairs are correctly matched
The Concurrent list comprises of 52 items
FN


including criminal and civil procedure, Supplementary Notes:
marriage and divorce, economic and
special planning trade unions, electricity, Zonal councils
D

newspapers, books, education, population  The idea of creation of Zonal Councils was
control and family planning etc.
.P

mooted by the first Prime Minister of India,


 Both the Parliament and the State Pandit Jawahar Lal Nehru in 1956 when
W

legislatures can make laws on subjects during the course of debate on the report
W

PPP-PTS-4273/082023/04 19
W
https://www.pdfnotes.co/

of the States Re-organisation Commission,  On the premise that strong states make
he suggested that the States proposed to a strong nation, NITI Aayog acts as the
be reorganised may be grouped into four or quintessential platform for the Government
five zones having an Advisory Council ‘to of India by bringing States together as
develop the habit of cooperative working” ‘Team India’ to work towards the national
among these States. development agenda.
 This suggestion was made by Pandit Nehru  In view of this, a number of steps have been
at a time when linguistic hostilities and taken by NITI Aayog to foster cooperative
bitterness as a result of re-organisation federalism through structured support
of the States on linguistic pattern were initiatives and engagement with the States/
threatening the very fabric of our nation. UTs on a continuous basis.
 As an antidote to this situation, it was  These include meetings between the Prime
suggested that a high level advisory forum Minister/Cabinet Ministers and all Chief
should be set up to minimise the impact Ministers; subgroups of Chief Ministers
of these hostilities and to create healthy on subjects of national importance;
inter-State and Centre-State environment sharing of best practices; policy support
with a view to solving inter-State problems and capacity development of State/UT
and fostering balanced socio economic
functionaries; launching of the Aspirational
development of the respective zones.

RE
Districts Programme for development of
 In the light of the vision of Pandit Nehru, backward districts; theme-based extensive
five Zonal Councils were set up vide Part-III engagements in various sectors; framing
of the States Re-organisation Act, 1956. The model laws for land leasing and agriculture
present composition of each of these Zonal marketing reforms; and area-specific
Councils is as under: interventions for the North-Eastern and
 The Northern Zonal Council, Himalayan States and island development.
comprising the States of Haryana,
Himachal Pradesh, Jammu &
O  NITI Aayog has been providing relevant
technical advice to the Centre, States and
Kashmir, Punjab, Rajasthan, UTs. NITI has also established models
National Capital Territory of Delhi and programmes for the development
and Union Territory of Chandigarh. of infrastructure and to reignite and
SC
 The Central Zonal Council, establish private-public partnership,
comprising the States of such as the Centre-state partnership
Chhattisgarh, Uttarakhand, Uttar model Development Support Services
Pradesh and Madhya Pradesh. to States and Union Territories
(DSSS); and the Sustainable Action for
 The Eastern Zonal Council, Transforming Human Capital (SATH)
comprising the States of Bihar, programme.
Jharkhand, Orissa, Sikkim and West
Bengal. L3: Real Challenge
GS

 The Western Zonal Council,


comprising the States of Goa, 41. Correct option: (b)
Gujarat, Maharashtra and the
Union Territories of Daman & Diu Explanation:
and Dadra & Nagar Haveli.  Both statements are correct
O
 The Southern Zonal Council, comprising
Emergency provisions
.C
the States of Andhra Pradesh,
Karnataka, Kerala, Tamil Nadu and the  The term “emergency” refers to an
S

Union Territory of Puducherry. unanticipated turn of circumstances


that requires government bodies to act
TE

L3: Real Challenge


immediately within their jurisdiction.
 In situations of emergency, all civil
40. Correct option: (d)
O

rights—except Articles 20 and 21 of


Explanation: the Constitution of India suspended
FN

inside the state or nation. The failure


 Both statements are correct of administrative equipment causes the
Cooperative federalism majority of emergencies.
D

 NITI Aayog has been constituted  According to Dr Bhimrao Rao Ambedkar,


.P

to actualise the important goal of the Indian federal structure is distinctive


cooperative federalism and to enable in that it can become unitary when the
W

good governance in India. institutional system breaks down.


W

20 PPP-PTS-4273/082023/04
W
https://www.pdfnotes.co/

 Additionally, the main goal of emergency  It introduced a new chapter (Part


legislative measures was to protect the IVA) in the Constitution that outlines
area from dictatorship combined with the the Fundamental Duties of Indian
domestic disorder, conflicts, and foreign citizens.
invasions.
 These are moral and civic responsibilities to
L2: Approachable the nation, such as respecting the national
flag, defending the country, promoting
harmony, etc.
42. Correct Option: (b)
 The amendment accorded precedence to
Explanation:
Directive Principles over Fundamental
 Statement 1 is incorrect: Article 352 Rights, stating that if there is a conflict
states that the President can proclaim an between the two, the Directive Principles
emergency when the territory of India is would take precedence.
under invasion, foreign encroachment, or
 The amendment extended the duration of
internal insurrection.
the President’s rule from one year to three
Supplementary Notes: years during a national emergency.

National Emergency The amendment declared that

RE

Parliament had the power to amend
 Even though such a proclamation could
any part of the Constitution, including
only be issued in the legislative chamber’s
Fundamental Rights, and this power
attendance and with each house’s consent,
could not be challenged in court.
the emergency was revoked after one
month.  It altered the balance between the Center
and the States by granting greater control
 The nation’s first emergency was
proclaimed during the 1962–1968
O
confrontation with China. Following it,
Smt. 
to the Union government, thereby reducing
the autonomy of the States.
The amendment abolished the privy purses
 Indira Gandhi imposed the most and privileges granted to the former rulers
SC
controversial state of emergency due to of princely states by the 26th Amendment
domestic strife. Act.

L1 : Elementary L3: Real Challenge

43. Correct option: (b) 44. Correct option: (c)


Explanation: Explanation:
GS

 Statement 3 is incorrect: The amendment  All Statements are correct


declared that Parliament had the power
Supplementary Notes:
to amend any part of the Constitution,
including Fundamental Rights, and this Financial Emergency
power could not be challenged in court. O
 The Constitution of India talks about three
Supplementary Notes: types of emergencies; National Emergency
.C
in Article 352, President’s Rule in State is
42nd Amendment Act
mentioned under article 356 and Financial
S

 The 42nd Amendment Act in India is a Emergency in Article 360 of the Indian
significant constitutional amendment that Constitution.
TE

was passed in 1976 during the period of


emergency in the country.  Part XVIII of the Constitution of India
contains emergency provisions from Articles
O

 It made several changes to the Indian 352 to 360.


Constitution and is considered one of the
FN

most controversial amendments in Indian  If the President of India is satisfied that


history. a situation has arisen due to which the
financial stability or credit of India or any
D

 The amendment changed the Preamble


part of its territory is threatened.
of the Indian Constitution to include the
.P

terms “Socialist,” “Secular,” and “Integrity”  He/she can declare the Financial Emergency
to reflect the ideals and aspirations of the on the aid and advise of the Council of
W

nation. Ministers.
W

PPP-PTS-4273/082023/04 21
W
https://www.pdfnotes.co/

 Article 360 gives authority to the  Part-time members: Maximum of 2


President of India to declare a financial members from foremost universities,
emergency. leading research organizations, and
other innovative organizations in an ex-
 But keep in mind that the 44th Constitutional officio capacity. Part-time members will
Amendment Act of 1978 says that the be on a rotational basis.
President’s ‘satisfaction’ is not beyond
judicial review. It means the Supreme Supplementary Notes:
Court can review the declaration of
NITI Aayog
Financial Emergency.
 The NITI Aayog was formed on January 1,
Parliamentary Approval and Duration of 2015. In Sanskrit, the word “NITI” means
the Financial Emergency morality, behaviour, guidance, etc. But, in
 A proclamation of financial emergency the present context, it means policy and the
must be approved by both the Houses of NITI stands for “National Institution for
Parliament within two months from the Transforming India”.
date of its issue.  The NITI Aayog is based on the 7 pillars of
 Once approved by both the Houses of effective Governance. They are:
Parliament, the Financial Emergency  Pro-people: it fulfils the aspirations of
continues indefinitely till it is revoked. This

RE
society as well as individuals
implies two things:
 Pro-activity: in anticipation of and response
 Repeated Parliamentary approval is not to citizen needs
required for its continuation.
 Participation: involvement of the citizenry
 There is no maximum time limit
 Empowering: Empowering, especially
prescribed for the operation of financial
women in all aspects
emergency

O
Resolution approving the proclamation of
 Inclusion of all: inclusion of all people
irrespective of caste, creed, and gender
financial emergency can be passed by either
House of Parliament (Lok Sabha or Rajya  Equality: Providing equal opportunity to all
Sabha) only by a simple majority. especially for youth
SC
L3: Real Challenge  Transparency: Making the government
visible and responsive

45. Correct Option: (b) L3: Real Challenge

Explanation:
46. Correct Option: (c)
 Statement 1 is incorrect: Governing
Council of NITI Aayog consists of the Explanation:
GS

Chief Ministers of all the States and  Statement 1 is correct: The National
Lt. Governors of Union Territories in Human Rights Commission, India has been
India. set up by an Act of Parliament under the
 Regional Councils will be created to address Protection of Human Rights Act, 1993 for
particular issues and possibilities affecting the protection and promotion of human
rights.
more than one state. These will be formed
O
for a fixed term. It will be summoned by the  The functions of the Commission as
.C
Prime Minister. stated in Section 12 of the Act and apart
from enquiry into complaints of violation
 It will consist of the Chief Ministers of States
S

of human rights or negligence in the


and Lt. Governors of Union Territories.
prevention of such violation by a public
TE

These will be chaired by the Chairperson of


servant, the Commission also studies
the NITI Aayog or his nominee.
treaties and international instruments on
 Statement 2 is correct: NITI Aayog human rights and make recommendations
O

replaced the Planning Commission of India. for their effective implementation to the
It is basically a think-tank or an advisory Government.
FN

body. The Planning Commission designed  Statement 2 is correct: The Commission


Five Years Plans in India. is responsible for spreading of human
D

 Statement 3 is correct: The full-time rights awareness amongst the masses and
organizational framework will include, encouraging the efforts of all stake holders
.P

in addition to the Prime Minister as the in the field of human rights literacy not only
Chairperson: Vice-Chairperson (appointed at the national level but at international
W

by the Prime Minister) level too.


W

22 PPP-PTS-4273/082023/04
W
https://www.pdfnotes.co/

 NHRC is a unique institution because it Supplementary Notes:


is one of the few National Human Rights
Institutes (NHRIs) in the world whose State Information Commission Powers
Chairperson is the former Chief Justice of and Functions
the country.  Any person may file a complaint with the
Commission, which must consider it. If there
Supplementary Notes:
are sufficient grounds, the Commission may
National Human Rights Commission order an inquiry into any topic (suo moto
(NHRC) power).
 The NHRC, India plays an active role in  The Commission can conduct investigations
coordinating with other NHRIs of the world in civil cases with the same authority as a
to enhance awareness from the perspective of civil court.
human rights. It has also hosted delegations  Any record controlled by a public authority
from UN Bodies and other National Human may be examined by the Commission during
Rights Commissions as well as members of the investigation of a complaint, and no
civil society, lawyers and political and social such record may be withheld from it for any
activists from many countries. reason.
The National Human Rights Commission

RE
  In other words, during the investigation,
(NHRC) of India was established on 12 all public records must be provided to the
October, 1993. The statute under which it Commission for review. The Commission
is established is the Protection of Human has the authority to ensure that the public
Rights Act (PHRA), 1993 as amended by the authority abides by its rulings.
Protection of Human Rights (Amendment)
Act, 2006. L1 : Elementary
It is in conformity with the Paris Principles,

O
adopted at the first international workshop
on national institutions for the promotion
and protection of human rights held in
48. Correct Option: (c)
Explanation:
Paris in October 1991, and endorsed by the  Statement 1 is incorrect: It is empowered
SC
General Assembly of the United Nations to inquire into offences alleged to have
by its Regulations 48/134 of 20 December, been committed under the Prevention of
1993. Corruption Act, 1988 by certain categories
 Section 2(1) (d) of the PHR Act defines of public servants.
Human Rights as the rights relating to  Statement 2 is incorrect: The IEMs will
life, liberty, equality and dignity of the examine all complaints received by them
individual guaranteed by the Constitution and give their recommendations to the chief
GS

or embodied in the International Covenants executive of the organisation. In case of


and enforceable by courts in India. suspicion of serious irregularities requiring
L2: Approachable legal or administrative action, they are
supposed to send a report to the Chief
Vigilance Officers (CVOs), the rules state.
47. Correct Option: (c) O
Supplementary Notes:
Explanation:
.C
Central Vigilance commission (CVC)
 Statement 1 is correct: According to the
 The Special Police Establishment (SPE)
Right to Information Act, 2005, states
S

which was set up in 1941 by the Government


in India are required to create a State
TE

of India.
Information Commission, which must be
published in the state’s official gazette.  The functions of the SPE then were to
investigate cases of bribery and corruption
O

 The State Information Commission is a


in transactions with the War & Supply
strong, independent body that conducts
Department of India during World War II.
FN

investigations and renders judgments


on appeals.  Even after the end of the War, the need for a
Statement 2 is incorrect: Under the Central Government agency to investigate
D


terms of the Right to Information Act of cases of bribery and corruption by Central
Government employees was felt.
.P

2005, the state government formed the


State Information Commission, a quasi-  The Delhi Special Police Establishment Act
W

judicial entity. was therefore brought into force in 1946.


W

PPP-PTS-4273/082023/04 23
W
https://www.pdfnotes.co/

 After promulgation of the Act,  One hundred percent of Lok Sabha members
superintendence of SPE was transferred might summon a conference to debate
to the Home Department and its functions repealing the proclamation.
were enlarged to cover all departments of
the Government of India.  It must be made within 14 days. An easy
majority vote at the special meeting will put
 The jurisdiction of SPE was extended to all
an end to the crisis.
the Union territories and the Act provided
for its extension to States with the consent  In most cases, a state of emergency lasts
of the State Government. a year. The 44th Amendment to the U.S.
 By 1963, the SPE was authorized to Constitution, passed in 1978, safeguards
investigate offences under 91 different this freedom.
sections of Indian Penal Code (IPC) and 16  After the 44th Amendment Act was signed
other Central Acts besides offences under
into law in 1978, writs of habeas corpus may
the Prevention of Corruption Act 1947.
be filed with the Supreme Court and the
L2: Approachable High Courts. Constitutional Amendment
352 (38th Amendment) rendered this
49. Correct option: (b) unjusticiable.

RE
 As of today, anybody may challenge any
Explanation:
declaration of emergency in court based
 Statement 3 is incorrect: The National on the government’s bad intentions,
Emergency does not stop the enforcement of
thanks to the removal of Clause 5. The
Articles 20 and 21 rights.
National Emergency does not stop
Supplementary Notes: the enforcement of Articles 20 and 21
O rights.
44th Amendment Act
L3: Real Challenge
 The 42nd Amendment Act, passed in 1976,
overrode the will of the Indian people by
SC
amending the constitution. 50. Correct option: (a)
 Article 352 was used by Union
Explanation:
Government to declare the Emergency.
Fortunately, the 1978 Constitution Act set  Option (a) is correct
out to correct these flaws and reestablish
harmony between government and people. Supplementary Notes:
This article goes into great detail on India’s
National Investigation Agency
GS

44th Constitutional Amendment.


 The National Investigation Agency (NIA)
 In both cases, the government proclaims a
state of emergency, and the two occurrences in India is a federal agency responsible
are almost similar. for investigating and combating terrorism
and other serious crimes with a national or
A two-thirds majority is needed to pass

cross-border impact. It was established on
O
the 44th Amendment of 1978. To make a
December 31, 2008, through the enactment
.C
proclamation lawfully, two-thirds of the
of the National Investigation Agency Act
House and Senate must support it.
2008.
S

 It would have expired after two months had


The primary objective of the NIA is to
TE

no action been taken if neither chamber had 


approved. Before the 44th Amendment was handle terror-related incidents and other
signed into law, it was little over a month. significant offenses that pose a threat to the
O

sovereignty, integrity, and security of India.


 After the emergency declaration was
The agency operates under the Ministry
FN

accepted by both houses of parliament in


1975, no further review of it was necessary. of Home Affairs, and its headquarters are
located in New Delhi.
The 44th Amendment Act of 1978
D


required a six-month review of the  Counter-terrorism operations:
.P

emergency declaration and, without Investigating and prosecuting acts of


new parliamentary assent, its terrorism, including attacks, bomb blasts,
W

termination after that period. hijackings, and other related activities.


W

24 PPP-PTS-4273/082023/04
W
https://www.pdfnotes.co/

 Probing cross-border crimes:  Handling organized crime: Investigating


Investigating offenses that have and prosecuting organized crime syndicates
international ramifications or involve other involved in activities such as drug trafficking,
countries. arms smuggling, human trafficking, and
 Dealing with counterfeit currency more.
cases: Investigating and prosecuting cases
 Protection of witnesses and informants:
related to the circulation of fake currency
notes. Ensuring the safety and security of
witnesses and informants during the course
 Combating cybercrimes: Addressing of investigations.
cyber threats and conducting investigations
related to cybercrime. L1 : Elementary

™™™™™

RE
O
SC
GS

O
S .C
TE
O
FN
D
.P
W
W

PPP-PTS-4273/082023/04 25
W

You might also like